This ladder is extended to a length of 18 feet. The bottom of the ladder is 4. 5 feet from the base of the building. What angle does the ladder make with the ground?

Answers

Answer 1

The ladder makes an angle of approximately 82 degrees with the ground when it is extended to a length of 18 feet and the bottom of the ladder is 4.5 feet from the base of the building.

To determine the angle that the ladder makes with the ground, we can use trigonometry. Let x be the height of the ladder when it is leaned against the building. Then, using the Pythagorean theorem, we have: [tex]x^{2}[/tex] + [tex]4.5^{2}[/tex] = [tex]18^{2}[/tex]

Solving for x, we get: x = sqrt([tex]18^{2}[/tex] - [tex]4.5^{2}[/tex]), x ≈ 17.29

Therefore, the ladder makes an angle θ with the ground such that: sin θ = opposite/hypotenuse = x/18, θ = arcsin(x/18)

Substituting x ≈ 17.29, we get: θ ≈ 81.99 degrees

Therefore, the ladder makes an angle of approximately 82 degrees with the ground when it is extended to a length of 18 feet and the bottom of the ladder is 4.5 feet from the base of the building.

To know more about Pythagorean theorem, refer here:

https://brainly.com/question/14930619#

#SPJ11


Related Questions


the girl lifts a painting to a height of 0.5 m in 0.75 seconds. how much
power does she use? *

Answers

Power is the rate at which work is done or energy is transferred. In this case, the girl used a force of 98 N to lift the painting to a height of 0.5 m in 0.75 seconds, resulting in 49 J of work done. The power used was calculated to be approximately 65.33 watts.

To calculate the power used by the girl while lifting the painting, we need to use the formula: Power (P) = Work (W) / time (t).

Firstly, we need to calculate the work done by the girl in lifting the painting. Work is defined as the product of force and distance. As there is no information about the force applied, we will assume that the girl lifted the painting with a constant force. Therefore, the work done can be calculated as:

Work (W) = force x distance

Here, the distance is 0.5 m, and we can use the formula for weight to calculate the force required to lift the painting. As we know that the mass of the painting is not given, we can assume it to be 10 kg (a medium-sized painting).

Weight (Wt) = mass x acceleration due to gravity
Wt = 10 kg x 9.8 m/s² = 98 N

Therefore, the work done by the girl is:

W = 98 N x 0.5 m = 49 J

Now, we can use the formula for power to calculate the power used by the girl.

P = W / t
P = 49 J / 0.75 s
P = 65.33 W (approx.)

Therefore, the girl used approximately 65.33 watts of power while lifting the painting.

You can learn more about Power at: brainly.com/question/30697721

#SPJ11

The girl used 65.3 watts of power to lift the painting.

How to find power?

To calculate power, we need to know the work done and the time taken.

We can use the formula:

power = work/time

The work done is equal to the force applied multiplied by the distance moved. Since we don't know the force, we can use the formula for work in terms of mass, gravity, and height:

work = mgh

where m is the mass, g is the acceleration due to gravity, and h is the height lifted.

Assuming the painting has a mass of 10 kg and the acceleration due to gravity is 9.8 m/s², the work done is:

work = (10 kg) x (9.8 m/s²) x (0.5 m) = 49 J

The time taken is 0.75 seconds.

So the power used is:

power = work/time = 49 J / 0.75 s = 65.3 watts

Therefore, the girl used 65.3 watts of power to lift the painting.

Learn more about power

brainly.com/question/23731425

#SPJ11

PLEASE HELP ASAP 3 PART QUESTION

Answers

Answer:

that is really hard but im pretty sure one of the answers to the first one is -16? for the second x

Step-by-step explanation:

A rental car company charges $22. 15 per day to rent a car and $0. 07 for every mile driven. Aubrey wants to rent a car, knowing that:


She plans to drive 275 miles.


She has at most $130 to spend.



Write and solve an inequality which can be used to determine dd, the number of days Aubrey can afford to rent while staying within her budget

Answers

An inequality to represent this situation is 22.15d + 0.07(275) ≤ 130. Aubrey can afford to rent the car for up to 5 days while staying within her budget.

Let's denote the number of days Aubrey can rent the car as "d". We know that the rental car company charges $22.15 per day and $0.07 per mile. Aubrey has a budget of $130 and plans to drive 275 miles. We can create an inequality to represent this situation:

22.15d + 0.07(275) ≤ 130

Now, let's solve the inequality:

22.15d + 19.25 ≤ 130

Subtract 19.25 from both sides:

22.15d ≤ 110.75

Now, divide by 22.15 to find the maximum number of days Aubrey can rent the car:

d ≤ 110.75 / 22.15

d ≤ 5

So, Aubrey can afford to rent the car for up to 5 days while staying within her budget.

More on inequalities: https://brainly.com/question/31624881

#SPJ11

the asq (american society for quality) regularly conducts a salary survey of its membership, primarily quality management professionals. based on the most recently published mean and standard deviation, a quality control specialist calculated the z-score associated with his own salary and found it was -2.50. this tells him that his salary is

Answers

This tells him that his salary is significantly below the average salary of quality management professionals surveyed by the ASQ, and that he is in the bottom percentile of salaries in this group.

The z-score is a statistical measure that indicates the number of standard deviations that a data point is from the mean of a distribution. A negative z-score indicates that the data point is below the mean.

In this case, the quality control specialist's z-score of -2.50 indicates that his salary is 2.50 standard deviations below the mean salary of the quality management professionals surveyed by the ASQ.

Without knowing the specific mean and standard deviation provided by the survey, it is difficult to determine the exact value of the specialist's salary. However, we can use the z-score to estimate the percentile rank of his salary compared to the rest of the survey respondents.

Using a standard normal distribution table, we can see that a z-score of -2.50 corresponds to a percentile rank of approximately 0.0062 or 0.62%. This means that only about 0.62% of quality management professionals surveyed by the ASQ earn a salary lower than that of the quality control specialist.

Learn more about z-score

https://brainly.com/question/15016913

#SPJ4

Mr. Rogers recorded the height of 15 students from two of his classes. Based on these samples, what generalization can be made? The median student height in Class A is equal to the median student height in Class B. The range of the student heights in Class A is greater than the range of the student heights in Class B. The mean student height in Class A is less than the mean student height in Class B. The median student height in Class A is more than the median student height in Class B

Answers

"The median student height in Class A is equal to the median student height in Class B."

Based on the given information, we can conclude that the median student height in Class A is e.

qual to the median student height in Class B. However, we cannot make any definitive conclusions about the range or mean heights of the two classes based on this limited information.

The range is a measure of the spread of the data and is calculated by subtracting the minimum value from the maximum value. Without knowing the actual height values for each student in both classes, we cannot compare the ranges and determine which class has a greater range.

The mean height is a measure of the central tendency of the data and is calculated by adding up all the heights and dividing by the total number of students. Again, without knowing the actual height values, we cannot calculate the mean heights for each class and compare them.

Therefore, the only conclusion that can be made based on the given information is that the median student height in Class A is equal to the median student height in Class B.

To learn more about central tendency visit: https://brainly.com/question/30218735

#SPJ11

If a ball is dropped on the ground from a height of h m, then the ball reaches the ground with the
velocity V=4.43√h m/sec. Find the velocity with which a ball reaches the ground when it is dropped
from a height of 64 m.

Answers

The velocity with which a ball reaches the ground when it is dropped

from a height of 64 m is 35.44m/sec

How to determine the value

From the information given, we have that the equation representing the velocity of the ball is expressed as;

V = 4.43√h

Given that the parameters of the formula are;

V is the velocity of the ball from he ground.h is the height of the ball.

Since the height of the ball from the ground is 64m, we have to substitute the value, we have;

V = 4.43√64

Find the square root of the value

V= 4.43(8)

Now, multiply both the values to determine the velocity, we get;

V = 35.44m/sec

Learn about velocity at: https://brainly.com/question/25749514

#SPJ1

For the following function, find the Taylor series centered at 4 and give the stronger terms of the Taylor series Wite the intervat of convergence of the series (+) = In(1) (t)= Σ ร f(x) + The welval of convergence is (Give your answer in interval notation)

Answers

The Taylor series centered at 4 for f(x) = ln(1+x) is: f(x) = ln(5) + (x-4)/5 - (x-4)^2/50 + (2/125)*(x-4)^3 - (6/625)*(x-4)^4 + ... The interval of convergence for this series is (-∞, ∞).

Let's find the Taylor series centered at 4 for the function f(x) = ln(1+x).

We can use the formula for the Taylor series coefficients:

f^(n)(x) = (-1)^(n-1) * (n-1)! / (1+x)^n

where f^(n)(x) denotes the nth derivative of f(x).

Using this formula, we can find the Taylor series centered at 4: f(4) = ln(1+4) = ln(5) f'(x) = 1/(1+x), so f'(4) = 1/5 f''(x) = -1/(1+x)^2, so f''(4) = -1/25 f'''(x) = 2/(1+x)^3, so f'''(4) = 2/125 f''''(x) = -6/(1+x)^4, so f''''(4) = -6/625 and so on.

Putting it all together, the Taylor series centered at 4 for f(x) is:

f(x) = ln(5) + (x-4)/5 - (x-4)^2/50 + (2/125)*(x-4)^3 - (6/625)*(x-4)^4 + ...

To find the interval of convergence, we can use the ratio test:

lim |(f^(n+1)(x) / f^(n)(x)) * (x-4)/(x-4)| = lim |(-1) * (n+1) * (1+x)^2 / (1+x)^n| * |x-4| = lim (n+1) * (1+x)^2 / (1+x)^n * |x-4| = lim (n+1) / (1+x)^(n-2) * |x-4|

Since this limit is zero for all values of x, the interval of convergence is the entire real line, (-∞, ∞).

So the final answer is: The Taylor series centered at 4 for f(x) = ln(1+x) is: f(x) = ln(5) + (x-4)/5 - (x-4)^2/50 + (2/125)*(x-4)^3 - (6/625)*(x-4)^4 + ... The interval of convergence for this series is (-∞, ∞).

Learn more about Taylor series,

https://brainly.com/question/28168045

#SPJ11

The sides of the base of a right square pyramid are 3 meters in length, and its slant height is 6 meters. if the lengths of the sides of the base and the slant height are each multiplied by 3, by what factor is the surface area multiplied?

a. 12

b. 3^3

c. 3^2

d. 3

Answers

If the base and slant height both are divided by a factor of 3, the surface area will get multiplied by factor, option b, 3².

Here we are given that the square pyramid has a base of 3m and a slant height of 6 m.

The surface area formula for a square pyramid with square edge a and slant height h is

a² + 2a√(a²/4  + h²)

Here, a = 3 and h = 6. Hence we get

3² + 2X3√(3²/4  + 6²)

= 46.108

Now the base and slant height are multiplied by 3. Hence we will get

9a² + 6a√(9a²/4  + 9h²)

414.972

Now, dividing both obtained we will get

414.972/46.108

= 9

= 3²

Hence, it should be multiplied by 3².

To learn more about Surface Area visit

https://brainly.com/question/29298005

#SPJ4

A triangular prism is 40 yards long and has a triangular face with a base of 32 yards and a height of 30 yards. The other two sides of the triangle are each 34 yards. What is the surface area of the triangular prism?

Answers

The surface area of the triangular prism is 4800 square yard.

How to find the surface area of the triangular prism?

The surface area of a triangular prism is sum of the areas of the faces that make the prism.

The surface area of a triangular prism is given by:

SA = (a + b + c)L + bc

Where a and b are the bases of the rectangular faces, c is the height of the triangle and h is the total length of the prism

In this case:

L = 40, a = 34, b = 32 and c = 30

SA = (34 + 32 + 30)40 + (32 * 30)

SA = 3840 + 960

SA = 4800 square yard

Learn more about surface area on:

brainly.com/question/15777841

#SPJ1

7th Grade Advanced Math
Please answer my question no explanation is needed.
Marking Brainliest

Answers

The theoretical probability is: 12.5%.After 100 trials, the experimental probability is of: 20%.After 400 trials, the experimental probability is of: 11%.After more trials, the experimental probability is closer to the theoretical probability.

How to calculate a probability?

A probability is calculated as the division of the desired number of outcomes by the total number of outcomes in the context of a problem/experiment.

A probability can be classified as experimental or theoretical, as follows:

Experimental -> calculated after previous trials.Theoretical -> calculate before any trial.

The dice has eight sides, hence the theoretical probability of rolling a six is given as follows:

1/8 = 0.125 = 12.5%.

(each of the eight sides is equally as likely, and a six is one of these sides).

The experimental probabilities are obtained considering the trials, hence:

100 trials: 20/100 = 0.2 = 20%.400 trials: 44/400 = 0.11 = 11%.

The more trials, the closer the experimental probability should be to the theoretical probability.

More can be learned about probability at https://brainly.com/question/24756209

#SPJ1

SOMEONE HELP PLS!! giving brainliest to anyone!!

Answers

Answer:

252

Step-by-step explanation:

So their are 38 more numbers to get to 41 and the numbers are adding by 6, so mulitply 6 by 38 and you get 228 and add 228 to the biggest number of 24 and your final answer becomes 252.

WILL GIVE BRAINLIEST!!!


A team of scientists is studying the animals at a nature reserve. They capture the animals, mark them so they can identify each animal, and then release them back into the park. The table gives the number of animals they’ve identified. Use this information to complete the two tasks that follow.



Animal Total in Park Number Marked


elk 5,625 225


wolf 928 232


cougar 865 173


bear 1,940 679


mountain goat 328 164


deer 350 105


moose 215 86



What is the probability of the next elk caught in the park being unmarked? Write the probability as a fraction, a decimal number, and a percentage

Answers

The probability of the next elk caught in the park being unmarked can be calculated as follows:

There are a total of 5,625 elks in the park, out of which 225 have been marked.This means that the number of unmarked elks is 5,625 - 225 = 5,400.Therefore, the probability of the next elk caught in the park being unmarked is 5,400/5,625 = 0.96 or 96%.

What is the probability of capturing an unmarked elk at the park?

The probability of capturing an unmarked elk in a nature reserve park can be calculated by dividing the number of unmarked elks by the total number of elks.

In this case, the number of unmarked elks is 5,400 out of a total of 5,625 elks. This gives a probability of 96% or 0.96 in decimal form. Marking and tracking animals is a common method used by scientists to study animal populations in nature reserves.

This data is crucial for designing conservation strategies that promote the survival of endangered species. Nature reserves play a crucial role in preserving and protecting wildlife and their habitats, given the significant threats they face from habitat loss, poaching, and climate change.

Learn more about probability

brainly.com/question/30034780

#SPJ11

Solve: 5x + 6 > 3x + 15

Answers

Answer:

Subtract the smaller amount of [tex]x[/tex] → [tex]2x+6 > 15[/tex]

Then subtract 6 from 15 as it is a plus you do the opposite → [tex]2x > 9[/tex]

Now divide 9 by 2 to isolate [tex]x[/tex] → [tex]x > 4.5[/tex]

Help me please I need the answer to the value of x

Answers

5x+x+3x+4-2+7=180( straight line angles add up to 180)
9x+9=180
9x=171
x=19
Hope it helps

Which choice correctly compares two decimals?
A 2.17 > 2.0172.17 > 2.017
B 2.018 > 2.172.018 > 2.17
C 2.16 < 2.0172.16 < 2.017
D 2.17 = 2.017

Answers

Answer:

A

Step-by-step explanation:

2.17 > 2.017

because

2.017 = 2 + 17/1000

while

2.17 = 2 + 17/100 = 2 + 170/1000

170/1000 is larger than 17/1000.

for that reason D is wrong, of course.

2.17 is NOT equal to 2.017. 17/1000 is NOT equal to 170/1000.

2.018 = 2 + 18/1000

2.17 = 2 + 17/100 = 2 + 170/1000

also 18/1000 is NOT larger than 170/1000.

2.16 = 2 + 16/100 = 2 + 160/1000

2.017 = 2 + 17/1000

17/1000 are NOT larger than 160/1000.

Qn2. Two functions f and g are defined as follows: f(x) = 2x – 1 and g(x) = x +4. Determine: i) fg(x) ii) value of x such that fg(x) = 20

Answers

The value of x such that fg(x) = 20 is 6.5.

Find the value of f(x)g(x) by substituting g(x) into f(x):f(x)g(x) = f(x)(x+4) = 2x(x+4) - 1(x+4) = 2x^2 + 8x - 4

To find the composite function fg(x), we need to substitute the expression for g(x) into f(x), as follows:

fg(x) = f(g(x)) = f(x + 4) = 2(x + 4) - 1 = 2x + 7

So, fg(x) = 2x + 7

ii) To find the value of x such that fg(x) = 20, we can substitute fg(x) into the equation and solve for x, as follows:

fg(x) = 2x + 7 = 20

2x = 13

x = 6.5

Learn more about composite

brainly.com/question/13253422

#SPJ11

The function y=f(x) is graphed below. What is the average rate of change of the function f(x) on the interval −3≤x≤8?

Answers

The average rate of change of the function f(x) in the interval [tex]-3 \leq x\leq -2[/tex] is -15.

We are given an interval in which we have to find the average rate of change of the function f(x) based on the graph given in the question. The interval given is -3 [tex]\leq[/tex] x [tex]\leq[/tex] -2. We are going to apply the formula for an average rate of change to find the rate of change of the given function in the given interval.

The formula we will use is

The average rate of change = [tex]\frac{f(b) - f(a) }{b - a}[/tex]

Identifying the points in the graph,

a = 3, f(a) = -10

b = -2, f(b) = -25

We will substitute these values in the formula for the average rate of change.

The average rate of change = [tex]\frac{-25-(-10)}{-2-(-3)}[/tex]

The average rate of change = ( -25 + 10)/(-2 +3)

= -15/1

= -15.

Therefore, the average rate of change of the function in the interval [tex]-3 \leq x \leq -2[/tex] is -15.

To learn more about the average rate of a function;

https://brainly.com/question/2170564

#SPJ1

The complete question is "The function y=f(x)y=f(x) is graphed below. What is the average rate of change of the function f(x)f(x) on the interval -3\le x \le -2 −3≤x≤−2? "

A punch recipe calls for 1 1/2
quarts of sparkling water and
3/4 of a quart of grape juice.How much grape juice would you need to mix with
3 3/4 quarts of sparkling water?

Answers

Therefore, we need 3/4 of a quart of grape juice to mix with 3 3/4 quarts of sparkling water.

What is fraction?

In mathematics, a fraction represents a part of a whole or a ratio between two numbers. It consists of a numerator and a denominator, separated by a horizontal or diagonal line. The numerator is the number above the line and the denominator is the number below the line. The numerator and denominator can be any real numbers, including integers, decimals, or even other fractions.

Here,

The punch recipe requires a ratio of 1 1/2 quarts of sparkling water to 3/4 of a quart of grape juice. To determine how much grape juice is needed to mix with 3 3/4 quarts of sparkling water, we can set up a proportion:

1 1/2 quarts of sparkling water : 3/4 quart of grape juice = 3 3/4 quarts of sparkling water : x

To solve for x, we can cross-multiply and simplify:

(1 1/2) / (3/4) = (15/4) / (3/4)

= 15/3

= 5

3 3/4 * 1 / 5 = 15/20

= 3/4

To know more about fraction,

https://brainly.com/question/24638688

#SPJ1

PLEASE HELP!!!!!!!! The graph shows two lines, A and B. A coordinate plane is shown. Two lines are graphed. Line A has the equation y equals x minus 1. Line B has equation y equals negative 3 x plus 7. Based on the graph, which statement is correct about the solution to the system of equations for lines A and B? (4 points) Question 4 options: 1) (1, 2) is the solution to both lines A and B. 2) (−1, 0) is the solution to line A but not to line B. 3) (3, −2) is the solution to line A but not to line B. 4) (2, 1) is the solution to both lines A and B.

Answers

The correct statement regarding the solution to the system of equations is given as follows:

4) (2, 1) is the solution to both lines A and B.

How to solve the system of equations?

The system of equations in the context of this problem is defined as follows:

y = x - 1.y = -3x + 7.

Replacing the second equation into the first, the value of x is obtained as follows:

-3x + 7 = x - 1

4x = 8

x = 2.

Hence the value of y is given as follows:

y = 2 - 1

y = 1.

Meaning that point (2,1) is a solution to both lines.

More can be learned about a system of equations at https://brainly.com/question/13729904

#SPJ1

(1 point) Use the method of undetermined coefficients to find a solution of a y" – 8y' + 297 = 48e4t cos(3t) + 80e4t sin(3t) + 3 - Use a and b for the constants of integration associated with the homogeneous solution. Use a as the constant in front of the cosine term. y = yh + yp = - = (1 point) Find y as a function of x if ' y" – 6y" + 8y' = 3e", - - = y(0) = 14, y'(0) = 29, y"(0) = 25. 33 4x y(x) = 37 91 e2x - tet 8 e 8 4

Answers

By using the method of undetermined coefficients, The general solution is y = ae^(4x)cos(3x) + be^(4x)sin(3x) + (7/2cos(3t) + 5/2sin(3t))e^(4t). The solution to the initial value problem is y = 3e^(2x) + 14e^(4x) - 3e^(3x).

By using the method of undetermined coefficients, the associated homogeneous equation is y''-8y'+297=0, which has the characteristic equation r^2-8r+297=0. The roots of this equation are r=4+3i and r=4-3i, so the homogeneous solution is yh=a*e^(4x)cos(3x)+be^(4x)*sin(3x).

To find the particular solution, we make the ansatz yp = (Acos(3t) + Bsin(3t))e^(4t), where A and B are constants to be determined. Substituting this into the differential equation, we get

y" - 8y' + 297 = (16A - 18B)e^(4t)cos(3t) + (16B + 18A)e^(4t)sin(3t)

On the right-hand side, we have 48e^4tcos(3t) + 80e^4tsin(3t), which suggests setting

16A - 18B = 48, and

16B + 18A = 80

Solving these equations simultaneously, we get A = 7/2 and B = 5/2. Therefore, the particular solution is

yp = (7/2cos(3t) + 5/2sin(3t))e^(4t)

And the general solution is

y = yh + yp = ae^(4x)cos(3x) + be^(4x)sin(3x) + (7/2cos(3t) + 5/2sin(3t))e^(4t)

For the second problem, the associated homogeneous equation is y''-6y'+8y=0, which has the characteristic equation r^2-6r+8=0. The roots of this equation are r=2 and r=4, so the homogeneous solution is yh=ae^(2x)+be^(4x).

To find the particular solution, we make the ansatz yp = Ce^3x, where C is a constant to be determined. Substituting this into the differential equation, we get

y" - 6y' + 8y = 9Ce^3x - 18Ce^3x + 8Ce^3x = (8C - 9C)e^3x = -C*e^3x

On the right-hand side, we have 3e^x, which suggests setting -C = 3. Therefore, the particular solution is

yp = -3e^(3x)

And the general solution is

y = yh + yp = ae^(2x) + be^(4x) - 3e^(3x)

To find the values of a and b, we use the initial conditions

y(0) = a + b - 3 = 14

y'(0) = 2a + 4b - 9 = 29

y''(0) = 2a + 8b = 25

Solving these equations simultaneously, we get a = 3 and b = 14. Therefore, the solution to the initial value problem is

y = 3e^(2x) + 14e^(4x) - 3e^(3x)

To know more about method of undetermined coefficients:

https://brainly.com/question/31474346

#SPJ4

--The given question is incomplete, the complete question is given

"  (1 point) Use the method of undetermined coefficients to find a solution of a y" – 8y' + 297 = 48e4t cos(3t) + 80e4t sin(3t) + 3 - Use a and b for the constants of integration associated with the homogeneous solution. Use a as the constant in front of the cosine term. y = yh + yp = - = (1 point) Find y as a function of x if ' y" – 6y" + 8y' = 3e", - - = y(0) = 14, y'(0) = 29, y"(0) = 25."--

12 kilometers and the distance between the courthouse and the city pool is 15 kilometers, how far is the library from the community pool?

Answers

The library is approximately 19.2 kilometers from the community pool. The distance between the library and the community pool can be calculated using the Pythagorean theorem since the problem describes a right-angled triangle (due south and due west directions).

It is given that the distance between library and courthouse is 12 kilometers (south) and the distance between courthouse and community pool is 15 kilometers (west). Let's call the distance between the library and the community pool "x" kilometers.

According to the Pythagorean theorem:
a² + b² = c²

12² + 15² = x²

Now, calculate the square of the distances: 144 + 225 = x²
Add the numbers: 369 = x²

Finally, find the square root of the sum to find "x":
x = √369
x ≈ 19.2
The library is approximately 19.2 kilometers from the community pool.

To learn more about distance : https://brainly.com/question/26550516

#SPJ11

The value of P from the formula I= PRT/100 when I = 20, R= 5 and T= 4 is ?

Answers

The value of P from the formula  I= PRT/100 when I = 20, R= 5 and T= 4 is 100.The formula I = PRT/100 is used to calculate the simple interest on a principle amount, where P is the principle amount, R is the interest rate, and T is the time period.

To find the value of P from the formula I = PRT/100 when I = 20, R = 5, and T = 4,

Write down the formula: I = PRT/100 Plug in the given values: 20 = P(5)(4)/100Simplify the equation: 20 = 20P/100 Solve for P: P = 20(100)/20 = 100

Therefore the value of P is 100.

To learn more about value : https://brainly.com/question/843074

#SPJ11

Find the critical points c for the function / and apply the Second Derivative Test (if possible) to determine whether each of
these points corresponds to a local maximum (mar) or minimum (Gmin).
/(x) = 7x° In(3x) (* > 0)
(Use symbolic notation and fractions where needed. Give your answer in the form of a comma separated list, if necessary. Enter
DNE if there are no critical points.)
Cmin=
Cmax=

Answers

The critical points of f(x) are x = 0 and x = e^(-1/2) / 3, and x = e^(-1/2) / 3 corresponds to a local minimum of f(x). Cmin = e^(-1/2) / 3 and Cmax = 0.

Taking the derivative of f(x) with respect to x using the product rule and the chain rule, we get:

f'(x) = 14x ln(3x) + 7x

Setting f'(x) equal to zero and solving for x, we get:

14x ln(3x) + 7x = 0

Factor out x:

7x(2ln(3x) + 1) = 0

So either x = 0 or 2ln(3x) + 1 = 0.

If x = 0, then f'(x) = 0 and x is a critical point.

If 2ln(3x) + 1 = 0, then ln(3x) = -1/2 and 3x = e^(-1/2). Solving for x, we get:

x = e^(-1/2) / 3

So e^(-1/2) / 3 is also a critical point.

Now we need to apply the second derivative test to determine whether these critical points correspond to a local minimum or maximum.

Taking the second derivative of f(x), we get:

f''(x) = 14 ln(3x) + 21

For x = 0, we have:

f''(0) = 14 ln(0) + 21

The natural logarithm of zero is undefined, so the second derivative does not exist at x = 0. Therefore, we cannot apply the second derivative test at x = 0.

For x = e^(-1/2) / 3, we have:

f''(e^(-1/2) / 3) = 14 ln(1/e^(1/2)) + 21

= -14/2 + 21

= 7/2

Since the second derivative is positive at this point, we can conclude that x = e^(-1/2) / 3 is a local minimum of f(x).

To know more about critical point, here

https://brainly.com/question/31017064

#SPJ4

In a game show, players play multiple rounds to score points. Each round has 5 times
as many points available as the previous round.

Answers

An equation shows the number of points available, p, in round n of the game show is p=20·5ⁿ. Therefore, option D is the correct answer.

The given geometric sequence is 20, 100, 500, 2500,...

Here, a=20

Common ratio (r) = 100/20 = 5

The formula to find nth term of the geometric sequence is [tex]a_n=ar^n[/tex]. Where, a = first term of the sequence, r= common ratio and n = number of terms.

Here, aₙ=20·5ⁿ

Therefore, option D is the correct answer.

To learn more about the geometric sequence visit:

https://brainly.com/question/11266123.

#SPJ1

find the angle between the vectors. (round your answer to two decimal places.) u = (4, 3), v = (5, −12), u, v = u · v

Answers

The angle between u and v is approximately 104.66 degrees. To find the angle between two vectors u and v, we can use the dot product formula:

cos(theta) = (u · v) / (||u|| ||v||)

where ||u|| and ||v|| are the magnitudes of u and v, respectively.

First, let's compute the dot product of u and v:

u · v = [tex](4)(5) + (3)(-12) = 20 - 36 = -16[/tex]

Next, we need to find the magnitudes of u and v:

[tex]||u||[/tex] = sqrt([tex]4^2[/tex] + [tex]3^2[/tex]) = 5

[tex]||v||[/tex] = sqrt([tex]5^2[/tex] + (-12[tex])^2[/tex]) = 13

Now we can substitute these values into the formula for cos(theta):

cos(theta) = [tex](-16) / (5 * 13) = -0.246[/tex]

To find the angle theta, we take the inverse cosine of cos(theta):

theta = [tex]cos^-1[/tex](-0.246) = 104.66 degrees

Therefore, the angle between u and v is approximately 104.66 degrees.

Learn more about vectors ,

https://brainly.com/question/29740341

#SPJ4

If a, b and c are distinct real numbers, prove that the equation(x−a)(x−b)+(x−b)(x−c)+(x−c)(x−a=0has real and distinct roots.

Answers

Answer:

mail

Step-by-step explanation:

skating Dinero broke 1p revision yahoo d10

A table titled inequality symbols contains the symbols for less-than and greater-than.

Check all that are inequalities.

-3 = y

t > 0

-4. 3 < a

g = 5 and one-half

k less-than Negative Start Fraction 5 Over 7 End Fraction

x = 1

Answers

The inequalities in the given table are "t > 0" and "3 < a."

To identify the inequalities from the provided options, we need to understand the meaning of the symbols and check if they represent a comparison between two values.

-3 = y: This is not an inequality symbol but rather an equality symbol. It represents that -3 is equal to y, not greater or less than.

t > 0: This is an inequality symbol. The symbol ">" represents "greater than." Therefore, t is greater than 0.

-4. 3 < a: This is another inequality symbol. The symbol "<" represents "less than." Hence, 3 is less than a.

g = 5 and one-half: This is an equality symbol. The symbol "=" denotes equality, indicating that g is equal to 5 and one-half, not greater or less than.

k less-than Negative Start Fraction 5 Over 7 End Fraction: This is also an inequality symbol. The phrase "less than" indicates a comparison. The fraction "Negative Start Fraction 5 Over 7 End Fraction" represents -5/7. Therefore, k is less than -5/7.

x = 1: This is an equality symbol. The symbol "=" indicates that x is equal to 1, not greater or less than.

In summary, the inequalities in the table are "t > 0" and "3 < a."

To know more about inequalities , refer here :

https://brainly.com/question/30231190#

#SPJ11

Can someone help me with this question and show the steps please

Answers

Answer: [tex](w^{\frac{1}{5} } )^{3}[/tex]

Step-by-step explanation:

The root of a number, say [tex]\sqrt[n]{x}[/tex] is equal to [tex]x^{\frac{1}{n} }[/tex]. So, [tex]\sqrt[5]{w^{3} } = (w^{3} )^{\frac{1}{5} }[/tex]. Since when dealing with an exponent of a number raised to an exponent you multiply the exponents, due to the associative property it does not matter which order you do the exponents in. So, [tex](w^{3} )^{\frac{1}{5} }= (w^{\frac{1}{5} } )^{3}[/tex], which is answer D.

What is the sum of the series?
Σ (2k2 – 4)
k=1

Answers

The sum of the series Σ (2k² - 4) from k = 1 to n can be found using the following formula:

Σ (2k² - 4) = [2(1²) - 4] + [2(2²) - 4] + [2(3²) - 4] + ... + [2(n²) - 4]

= 2(1² + 2² + 3² + ... + n²) - 4n

The sum of squares of the first n natural numbers can be calculated using the formula:

1² + 2² + 3² + ... + n² = [n(n + 1)(2n + 1)] / 6

Substituting this value in the above equation, we get:

Σ (2k² - 4) = 2[(n(n + 1)(2n + 1)) / 6] - 4n

= (n(n + 1)(2n + 1)) / 3 - 4n

Therefore, the sum of the series Σ (2k² - 4) from k = 1 to n is (n(n + 1)(2n + 1)) / 3 - 4n.

An angle measures 11.4° more than the measure of its complementary angle. What is the measure of each angle?

Answers

The measure of the angle is 50.7° and the measure of its complementary angle is 39.3°.

What is the measure of each angle?

Let x be the measure of the angle and y be the measure of its complementary angle.

Then we have:

x = y + 11.4 (since the angle measures 11.4° more than its complementary angle)

x + y = 90 (since the two angles are complementary)

Substituting the first equation into the second equation, we get:

(y + 11.4) + y = 90

2y + 11.4 = 90

2y = 78.6

y = 39.3

Substituting y = 39.3 into the first equation, we get:

x = y + 11.4 = 50.7

So, we have

x = 50.7

Read more about angle at

https://brainly.com/question/28293784

#SPJ1

Other Questions
Using the drop-down menus below, identify the processes cells use to maintain homeostasis. A. is the movement of water along the concentration gradient. B. is the use of energy to move particles against the concentration gradient. C. is the movement of particles by diffusion without energy. D. is the movement of particles along the concentration gradient. The southern pine bark beetle lives in the bark of pine trees. The beetles feed on the tree and live within the bark of the tree, producing offspring. Once the beetles have infested a tree, they will spread to other trees in the same area. Eventually a pine tree infested by the beetle will die. The population of pine bark beetles can be expected to increase in a new habitat where it is introduced. The population would NOT be expected to decrease until what happens?Group of answer choicesAll the pine trees in the area have been killed off. Birds that feed on insects migrate away from the areaOther insects that feed on pine bark move out of the area. The pine trees in the area begin to produce seeds A man buys a car at a cost of r60 000 from cape town and transported it to durban at a cost price of by r4 500. at what price must he sell the car to make an overall profit of 25% How was education affected by the rule of the nazi party? schools were shut down and considered too liberal, so children joined aryan youth instead. the curriculum was shaped to teach nazi values and beliefs of racial superiority. teachers were trained by special propaganda, so parents called for their removal. school curriculum was changed to make jews and other minorities more "aryan." Please solve, I rate! :)Given f(t, y) = 22 4cy3 + 3y5, find 2. - f1(,y) fy(x, y) = = frz(, y) = fry(x, y) = after the civil war former slaves needed jobs and former slaveholders needed labor. explain how these systems met those needs. Question 11 of 25In the circuit below, resistors R and R are in parallel. What is the equivalent11resistance? (R = 30 22,R = 20 2) (A+A)Ptot RSSRS SwwR1Rk Help pls working and explanation needed When you learned the title of the artwork, how did you associate the words with what you see in front of you? did the title seem to fit with the artwork or were you surprised?(the farm by joan mir) a. What does the size of each section tell you about that portion of the data? Select all that apply.A. The relative importance of the categoryB. The difference between the minimum and maximum values within the categoryc. The count of data points within the categoryD. The relative frequency of data within the category PLEASE HELP! PHOTO ATTACHED Seth deposits X in an account today in order to fund his retirement. He would like to receive payments of 50 per year, in real terms, at the end of each year for a total of 12 years, with the first payment occurring seven years from now. The inflation rate will be 0. 0% for the next six years and 1. 2% per annum thereafter. The annual effective rate of return is 6. 3%. Calculate X 5. Kamal said that he can measurearea using squares that are 2 unitslong and 1 unit wide. What mistakedid Kamal make? The main span of a suspension bridge is the roadway between the bridges towers. The main span of the Walt Whitman Bridge in Philadelphia is 2000 feet long. This is 600 feet longer than two-fifths of the length of the main span of the George Washington Bridge in New York City. Write an equation to represent the given problem and solve it to find the length of the main span of the George Washington Bridge Which of the following best explains why a government might issuesubsidies?A. To promote government control of local industriesB. To increase the cost of imported goods, and increase tax incomeC. To offer assistance financially to offset the effects of foreigncompetitionD. To increase taxes a circular pool has a radius of 32 cm find its area? A 24. 59 g mixture of zinc and sodium is reacted with a stoichiometric amount of sulfuric acid. The reaction mixture is then reacted with 97. 7 mL of 4. 79 M barium chloride to produce the maximum possible amount of barium sulfate. Determine the percent sodium by mass in the original mixture. G how do you find the base of a rectangle if you only know the height Electric Field of DreamsPART A) To begin, click the Add button to add one object to the system. Observe the electric field around this charged object. You may move the object around the field by dragging it with your cursor. While the arrows indicate the direction of the electric field around the charge, the length of the arrows indicates the field strength. Based on your observations of the field, what is the charge on this object? Give your reasoning. PART B) Set the charged object in motion by dragging it and releasing it. What do you observe about the behavior of the field lines in the vicinity of the object?PART C) Add another charged object to the electric field by clicking the Add button again. What is the charge of this new object? Give your reasoning. What do you observe about the behavior of both the objects as well as the field lines in the vicinity of both the objects?PART D) Click the Remove button to remove one of these objects, and then click the Properties button to set properties for the next object you will add. Just change the sign of the charge to (+), then click Done. Click Add to add this new object to the field. Now what do you observe about the behavior of the two objects and the field lines that surround them?PART E) With the two oppositely-charged objects still in the field, apply an external field to the system: In the External Field box, simply drag the dot until it becomes an electric field vector in some direction. Observe, describe, and explain the behavior of the two objects What is TRUE about the career of Miles Davis? CA. His work "Kind of Blue" received critical acclaim. He was never offered any formal musical training. He is best known for his contribution to hot jazz. His song "Stardust was a commercial success. O C. OD. NJ